LSAT and Law School Admissions Forum

Get expert LSAT preparation and law school admissions advice from PowerScore Test Preparation.

 Administrator
PowerScore Staff
  • PowerScore Staff
  • Posts: 8950
  • Joined: Feb 02, 2011
|
#22809
Complete Question Explanation

Weaken. The correct answer choice is (E)

In this stimulus, the party spokesperson makes the argument that no net increase can result from the distribution of the $600 million surplus—either new taxes result, or the dismissal of workers. The correct answer choice will effectively attack the spokesperson's assessment of the situation.

Answer choice (A): This would not affect the strength of the assertion that there can be no net increase of spending to stimulate the province.

Answer choice (B): Whether the refund were to be issued in a lump sum or not has nothing to do with the central question of possibility of a net increase of in-province spending.

Answer choice (C): Like incorrect answer choice (B) above, this choice is completely irrelevant. There is no discussion, and no need to consider, whether taxpayers would be angry in response.

Answer choice (D): This choice does not weaken the assertion that there can be no net increase in local spending, so this answer choice is incorrect.

Answer choice (E): This is the correct answer choice. The spokesperson's assertion is that there can be no net increase in spending to stimulate the province's economy. But if there can be savings of expenditures outside the province, this could certainly lead to a net increase of money within the province.
 kim4
  • Posts: 3
  • Joined: Nov 02, 2013
|
#12297
Hi,

I've been doing very well in the logical reasoning weakening section, however I can't figure out Question 26 (page 3-85). I think that I am getting caught up in the passage, especially with the conclusion. I have no idea how to figure out if something would weaken the conclusion reached or not. I read through the HW help so I know the answer is E but I don't get why or why a few of the other answers were wrong.

A. ?
B. seems irrelevant, so I knocked that answer out
C. again, seems irrelevant
D. why isn't this an option?
E. Why is this the right answer?

Can you possibly rephrase the conclusion? I wasn't able to come up with an accurate prephrase because the terminology was throwing me off.

Thank you very much
Kim
 Adam Tyson
PowerScore Staff
  • PowerScore Staff
  • Posts: 5377
  • Joined: Apr 14, 2011
|
#12298
Hey Kim, good question, and I can understand your confusion in dealing with this somewhat complex argument. This one falls into a fairly common category of LR stimuli - it's a "some people say...but they're wrong" structure, in which the author tells us that some group of people holds a certain opinion, but then goes on to show us why that opinion is wrong. The main conclusion of these types of arguments is always some variation of "but they're wrong."

Here, the first sentence is a complicated way of saying "they're wrong" - the author tells us that the spending increase some folks imagine will happen is "an illusory benefit." That is, it's an illusion - it won't happen. He then goes on to tell us, in a long and involved way, that no matter how you slice up the money or who gets it, there will be no net increase in spending inside the Province.

That conclusion is what we need to focus on - no net increase in spending. To weaken it, we want to find an answer that says "oh yes there WILL be a net increase in spending (specifically, spending INSIDE the province - more money will stay here at home). Answer choice E provides that - if we can become more efficient and spend $600 million less OUTSIDE the Province (which will therefore pay for the tax refund) then any refunded money spent by taxpayers INSIDE the Province will be a net increase. The workers keep their jobs, spending what they always did because they are still getting their paychecks, and everyone who gets money back spends it at home in addition to whatever they were already spending.

Answer D does provide an alternative to the refund proposal, but it doesn't do anything to address the argument's main conclusion, that the refund proposal will not result in a net increase in spending. Offering an alternative does nothing to weaken the original plan, it just says "there is another way."

This one was tough! Just getting a grasp on the economics involved, and recognizing the importance of in-Province vs. out-of-Province spending, is a real task. It's easy to get turned around and lose track of the essential elements that you need to track.

I hope everything else is going well for you! Good luck in your continued studies.
 lbayliyeva@unm.edu
  • Posts: 24
  • Joined: Jun 15, 2014
|
#15403
I am struggling with understanding why choice E is the correct answer.

Why would increase in out-of-province expenditure stimulate economic activity? How is it relevant to weakening the argument that no net increase can result from the distribution of the $600 million surplus.

Also, answer choice D seems to be very similar to answer choice E except it offers another suggestion on how $600 million could be used. Why is this incorrect answer?

Thank you so much for your help in advance.
 Jon Denning
PowerScore Staff
  • PowerScore Staff
  • Posts: 907
  • Joined: Apr 11, 2011
|
#15405
Thanks for the question! The proposal in question is to refund $600 million to the province's tax payers, which would then purportedly be spent by them and stimulate the economy. The problem the spokesperson raises is that, since that $600 million must be accounted for (balanced) in the budget, the province would need to make it back somehow: either tax the very people who are receiving it and basically negating the proposal's point of giving them money, or fire province workers who are part of the group getting the refund (killing their incentive to spend their new cash, since they're now unemployed).

We're asked to find a reason why the spokesperson might be wrong. In other words, why the refund proposal might work without the consequences envisioned.

Answer choice E does that beautifully by suggesting no one would need to be fired if the workers could be used more efficiently, saving $600 million in the process. Of course, where that $600 million is saved specifically is totally irrelevant, so the fact that they mention out of province expenditures is designed to trip you up. It doesn't matter. All that matters is the $600 million refunded is balanced, and no one would be fired. So we've shown the spokesperson's concerns to be invalid.

Answer choice D goes directly against the proposal we're trying to support: refunding the $600 million! Remember, we want to show how refunding the money to tax payers could work without the problems listed, NOT an alternative to the refund itself.

Hope that clears things up!
 ellenb
  • Posts: 260
  • Joined: Oct 22, 2012
|
#16176
Dear Powerscore,

I seem to be confused as to what is happening in this question. So, I get that there is a 600 surplus, however I do not understand it further as to the decisions and why the right answer is the right answer.

Regards,

Ellen
 David Boyle
PowerScore Staff
  • PowerScore Staff
  • Posts: 836
  • Joined: Jun 07, 2013
|
#16198
ellenb wrote:Dear Powerscore,

I seem to be confused as to what is happening in this question. So, I get that there is a 600 surplus, however I do not understand it further as to the decisions and why the right answer is the right answer.

Regards,

Ellen
Hello ellenb,

I think this a a weaken, not so much a cause-and-effect, question. (This is the question 26 on page 3-85, is it not?) Anyway, the stimulus says that a $600 million refund is bad, since it'll have to be compensated for somewhere else, like new taxes. However, answer E says that if workers are used more efficiently, that can save $600 million. Therefore, those savings would exactly balance the amount of the refund, so there would be no problem.

Hope this helps,
David
 Johnclem
  • Posts: 122
  • Joined: Dec 31, 2015
|
#28573
Hi powerscore,
Even after reading the above explanations , I am having trouble understanding why D is wrong here.

If D is true , doesn't it show we can have an increase in the economy by spending? since we are spending by creating jobs and these jobs would mean new taxes on top of whatever taxes we have now. :-? :x :-? :0


1 - there needs to be a balance : we either have to come up with new taxes or else dismiss workers ( cut back).
There can be no restarting net increase in spending to stimulate the province's economy .

D- provide nice could, instead of refunding the money, stimulate its economy by redirecting its spending to use 600 million for construction projects creating jobs around the province.


Thank you
John
 Nikki Siclunov
PowerScore Staff
  • PowerScore Staff
  • Posts: 1362
  • Joined: Aug 02, 2011
|
#28578
Hey John,

Thanks for the question. The issue is whether the opposition party's proposal to refund $600M will lead to increased spending or not. The author believes that it will not. To weaken the argument, we need to show that a tax refund is actually compatible with increased spending.

Answer choice (D) suggests that we can do something entirely different with the money - spend it on infrastructure ("instead of refunding the money"). Yes, infrastructure spending might create jobs and ultimately increase spending, but this isn't the proposal advocated by the opposition party. Answer choice (D) does not show that a tax refund is actually compatible with increased spending; rather, it proposes an entirely different solution to the problem. As such, answer choice (D) actually strengthens the argument that the tax refund is a bad idea.

Hope this clears it up!

Thanks,
 avengingangel
  • Posts: 275
  • Joined: Jun 14, 2016
|
#29979
I see why E is right. But D is wrong because the conclusion of the argument specifically talks about how the refund proposal would not result in a net increase? I didn't understand that to be true, I thought it was saying that either way, there will not be a net increase. It seemed to me that D provided a possibility of a net increase, therefore weakening the conlusion. Can you please elaborate on answer choice D? Thanks !!


(for my own future reference: 3-105, #26)

Get the most out of your LSAT Prep Plus subscription.

Analyze and track your performance with our Testing and Analytics Package.